Search found 195 matches


640 (Q48, V29) Shocked!!

I still reeling from my GMAT experience. Its been two days and I am unable to get over the shock. I was a bit nervous to start with and that affected me in the quant section, where I started to take a school-boy approach of solving the problem studiously as opposed to tactically eliminating choices ...

by sankruth

Thu Jan 31, 2008 2:45 am
Forum: I just Beat The GMAT!
Topic: 640 (Q48, V29) Shocked!!
Replies: 7
Views: 2822

gabriel wrote:Well, dividend is paid on the face value of the stock but nowhere does it say in the question that 50$ is the face value, so I am going to say E.

PS: - Guys please attach the source of the question.
This question is from a doc in BTG with about 100+ difficult quant questions.

by sankruth

Wed Jan 30, 2008 8:05 am
Forum: Data Sufficiency
Topic: Shares - Dividend
Replies: 9
Views: 2420

I feel A is the correct answer. Consider a case of 100 workers (80 Low-Pay, 20 High-Pay) Since LP had the greatest increase lets say 8 new people joined, while HP had 4 new joinees So total - 112 (88/24). As you can see 88/112 is less than 80%, although more people joined. (A) neatly summarises this...

by sankruth

Mon Jan 28, 2008 9:14 am
Forum: Critical Reasoning
Topic: Inference
Replies: 19
Views: 6373

Yes, its C but can you please explain how?

by sankruth

Sun Jan 27, 2008 7:27 am
Forum: Problem Solving
Topic: GMAT Prep question
Replies: 5
Views: 1705

GMAT Prep question

GMAT Prep question. How to solve this question?

by sankruth

Sun Jan 27, 2008 1:26 am
Forum: Problem Solving
Topic: GMAT Prep question
Replies: 5
Views: 1705

DS - GMAT Prep

d is a positive integer and f is the product of the first 30 positive integers. What is the value of d?

1. 10^d is a factor of f

2. d > 6

by sankruth

Sun Jan 27, 2008 1:21 am
Forum: Data Sufficiency
Topic: DS - GMAT Prep
Replies: 4
Views: 1858

Simplifying the exponents you get...

2^[(x+y)^2 - (x-y)^2]

The exponent part can be ssimplified as follows:

[(x+y)+(x-y)].[(x+y)-(x-y)]

2x.2y = 4xy = 4 (B)

by sankruth

Sun Jan 27, 2008 1:04 am
Forum: Problem Solving
Topic: GMAT Prep Question Exponents
Replies: 10
Views: 5919

I would go with A

The rabbit population has increased as a rseult of the removal of X and the clearing of Y sounds concise

by sankruth

Sun Jan 27, 2008 12:50 am
Forum: Sentence Correction
Topic: GMATPrep 35
Replies: 5
Views: 1997

D - It strengthens the argument with the evidence that although equal proportion of people have committed the crime a lower proportion of people get convicted if they are defended by private lawyers

by sankruth

Sun Jan 27, 2008 12:37 am
Forum: Critical Reasoning
Topic: Defendents
Replies: 5
Views: 2544

IMO the answer is B.

LCM will have all the unique prime factors of both numbers n & t, which will include the greatest prime factor.

by sankruth

Sun Jan 27, 2008 12:28 am
Forum: Data Sufficiency
Topic: GMATPrep GCF and LCM
Replies: 5
Views: 2253

Thanks! I got mixed up with another idiom "so X as to Y".

by sankruth

Sat Jan 26, 2008 4:40 am
Forum: Sentence Correction
Topic: Congress debates...
Replies: 3
Views: 1616

Congress debates...

Congress is debating a bill requiring certain employers provide workers with unpaid leave so as to care for sick or newborn children. (A) provide workers with unpaid leave so as to (B) to provide workers with unpaid leave so as to (C) provide workers with unpaid leave in order that they (D) to provi...

by sankruth

Sat Jan 26, 2008 4:03 am
Forum: Sentence Correction
Topic: Congress debates...
Replies: 3
Views: 1616

Is the answer 0 < x < 2?

by sankruth

Fri Jan 25, 2008 11:42 am
Forum: Problem Solving
Topic: Kaplan question - stuck!
Replies: 9
Views: 1875

Look at it this way...

Lets say, x^4 = y

sqrt(y) = 9

So, y = 81 = x^4

So x can be +3 or -3

by sankruth

Fri Jan 25, 2008 8:25 am
Forum: Data Sufficiency
Topic: Square Roots
Replies: 3
Views: 1522

Its from the 1000SC set. I guess the OA must be wrong then!

by sankruth

Thu Jan 24, 2008 1:58 pm
Forum: Critical Reasoning
Topic: Ibora
Replies: 12
Views: 6331